Prueba de que el potencial de cuatro vectores es un cuatro vectores válido

La definición de A m que me han dado es como un objeto creado a partir del campo escalar ϕ y el campo vectorial A es decir:

A m = ( ϕ , A C )
[en unidades cgs]

¿Cómo pruebo formalmente que este es de hecho un cuatro vector válido y no un revoltijo de objetos?

(La prueba de que cuatro potenciales es una pregunta de cuatro vectores no responde a esto; plantea un caso específico extraño).

se sigue del hecho de que F m v se transforma como un tensor de segundo rango, estos se derivan de las ecuaciones de Maxwell
Este tipo de pregunta no se puede responder de forma genérica. Depende de las suposiciones de las que parte. Alguien podría elegir un marco lógico en el que la naturaleza cuadrivectorial del potencial sea uno de los postulados.

Respuestas (5)

Antecedentes: ¿cuáles son estos potenciales?

Para ver que este es un cuadrivector, primero debemos entender de dónde viene. Tenemos las ecuaciones de Maxwell, donde usaré X ˙ = w X = C 1 t X escribir como,

mi = 4 π ρ × mi = B ˙ B = 0 × B = 4 π C 1 j + mi ˙ ,
y después de usar B = 0 para definir una familia de A a través de B = × A y el consiguiente × ( mi + A ˙ ) = 0 para definir una familia de φ a través de mi = A ˙ φ , podemos deducir que las dos ecuaciones restantes establecen,
A ˙ φ ¨ = 4 π ρ ( A ) 2 A = 4 π C 1 j A ¨ φ ˙
que se puede reescribir fácilmente (definiendo X = X ¨ 2 X , y λ = φ ˙ + A ) como,
φ = 4 π ρ + λ ˙ , A = 4 π C 1 j λ .
Ahora podríamos haber elegido un conjunto diferente de potenciales y aún así obtener los mismos campos: sabemos que podemos agregar cualquier x a A y preservar B = × A porque el rotacional de un gradiente es cero; mirando lo que se necesita para preservar mi sólo nos dice que debemos restar simultáneamente x ˙ de φ para preservar mi . Esto no afecta directamente la forma de las dos últimas ecuaciones, pero mapea el valor de λ λ x , y entonces podemos esencialmente elegir una forma funcional diferente para λ , con el argumento de que si queremos, digamos, λ = φ ˙ (calibrador de Coulomb) podemos tomar cualquier otra solución ( A , φ ) y calcula su λ y luego resolver x = λ φ ˙ para encontrar un x lo que nos da un conjunto diferente de campos que tiene esta forma funcional.

Por supuesto, el indicador de Lorenz que ahora asumimos es para resolver para A , φ tal que λ = 0 , flexible,

φ = 4 π ρ A X = 4 π C 1 j X A y = 4 π C 1 j y A z = 4 π C 1 j z

Estoy pasando por todo esto por tres razones:

  1. Usted afirmó que su ecuación era válida en unidades CGS y creo que está equivocado,
  2. Destacar que aun cuando encuentras una solución no es única; otras soluciones α = 0 se puede agregar a cualquier componente;
  3. Para afirmar que aquí hay un orden claro: uno comienza conociendo las densidades de carga y corriente y luego resuelve estos campos, que pueden usarse para derivar el mi y B campos si uno lo desea.

Usa el hecho de que ( ρ , j / C ) es un cuatro vector.

La prueba aproximada de que ( ρ , j / C ) es un cuatro vector implica imaginar primero una distribución de carga estática: una distribución de carga estática ( ρ 0 , 0 ) se convierte ( γ   ρ 0 , γ   ρ 0 β ) bajo una transformada de Lorentz; esto se ve fácilmente como un ( ρ , j / C ) par. Pero si se trata de un cuatro vectores, entonces una disposición más compleja es un cuatro vectores, precisamente porque una distribución de corriente de carga arbitraria puede aproximarse arbitrariamente mediante una superposición de un grupo de pequeñas distribuciones de corriente de carga estáticas que han sido impulsadas de varias maneras. Si todos ellos se transforman individualmente de manera adecuada sobre las transformadas de Lorentz, entonces su suma también debe hacerlo, porque las transformadas de Lorentz son lineales.

si estas de acuerdo conmigo en que ( ρ , j / C ) es un cuatrivector entonces este también debe ser el caso para ( φ , A / C ) , también por un argumento de linealidad, aunque es una suficiencia y no una necesidad (como debe ser: el calibre de Lorenz no ha precisado al 100% los campos exactos φ y A , por lo que debe haber otros campos que no sean la transformada de Lorentz de estos campos que también pasen el calibre de Lorenz y sean válidos).

Simplemente examine las ecuaciones en el indicador de Lorenz, después de un impulso de Lorentz de los campos de carga por β en el z ^ dirección:

ϕ = 4 π   γ   ( ρ β C 1 j z ) A X = 4 π C 1   j X A y = 4 π C 1   j y A z = 4 π C 1   γ   ( j z β C ρ )
Claramente es suficiente resolver estas ecuaciones para encontrar A X = A X y A y = A y . Para A z claramente es suficiente tener A z = γ   A z γ β ϕ y para φ = γ   φ γ β A z .

Así: si has resuelto ( ρ , j / C ) para los campos ( φ , A ) en el calibre de Lorenz, entonces puede obtener una solución válida para los campos ( φ , A ) que obtendría resolviendo las mismas ecuaciones para la ecuación potenciada por Lorentz ( ρ , j / C ) simplemente por Lorentz-transformando los campos ( φ , A ) como un cuatro vector.

En otras palabras: hay formas de hacer electromagnetismo en las que ese par no es un cuadrivector, pero no hace daño suponer que lo es .

En mi opinión, el hecho de que la cantidad de 4 dimensiones j = ( C ρ , j ) es un cuadrivector transformado de Lorentz no se demuestra a través de la ecuación de continuidad. Pero, al aplicar la transformación de Lorentz a las ecuaciones de Maxwell y tratar de dejar su forma sin cambios (covariante), inevitablemente nos vemos obligados a definir esto en el nuevo marco de tal manera que finalmente obedezca a la transformación de Lorentz. En esta definición es necesario utilizar el cuadrivector de velocidad de Lorentz. Por favor, corríjame si estoy equivocado.
...para mí no olvides recordarte: debes intercambiar A m , j m en ecuacion A m = m 0   j m .
Gracias por el comentario sobre este último. En cuanto a lo primero, no creo que sea la persona adecuada para hablar más de esto ... Ya no soy un gran admirador de la visión física de "cómo se transforma bajo la transformación de coordenadas" de lo que hace un [ a , b ] tensor un tensor. Lo que diría es que probablemente haya una manera, si uno viene desde esa perspectiva, de cortocircuitar todo el debate... β sabemos que recuperamos la transformada galileana no relativista y creo que se puede argumentar que ρ y j mezclarse desde allí primero.
No estoy de acuerdo con el primer párrafo. Si F es una función que satisface F = 0 , entonces ( A m + d 0 m F ) = m 0 j m . Pero no podemos concluir de esto que A m + d 0 m F es un vector -- de hecho, no lo es.
@AccidentalFourierTransform Sí, es por eso que ya no soy un gran admirador de esa empresa de hacer cosas ... una vez que entré en geometría diferencial, quedó mucho más claro que cuando tienes algo que "obviamente" no es un tensor, como, decir, Γ     i j k , en realidad debe tener mucho cuidado con lo que es obvio porque esa entidad, de hecho, representa un tensor válido; la objeción es realmente que el tensor no tiene esa función de ser un símbolo de Christoffel después de una transformación de coordenadas. ¿Supongo que lo mismo es cierto para tu ejemplo?
Rechacé tu respuesta como incorrecta.
@Frobenius un segundo, creo que tengo un argumento más correcto en líneas similares que presentaría, menos tratando de usar una noción nebulosa de invariancia de Lorentz y más basado en la linealidad simple.
@Frobenius Creo que me gusta más este argumento, y creo que también aborda el punto de AccidentalFourierTransform.
Esta es una buena respuesta. Es una confirmación de mi conclusión: que el potencial 4 es un vector 4 es una suposición , como en mi respuesta a mi propia pregunta aquí: ¿El hecho de que jμ sea un vector 4 implica que Aμ es, como argumenta Feynman? .
Por cierto, alternativamente me gusta la prueba de que el 4-corriente ( ρ , j / C ) es un cuadrivector basado en el postulado de covarianza de las ecuaciones de Maxwell. Es decir, si aplicamos una transformación de Lorentz a las ecuaciones de Maxwell de mi , B , j , ρ en un marco inercial, entonces para que estas ecuaciones sean invariantes en forma (covariante)...
.....debemos definir de manera inevitablemente única las nuevas cantidades mi , B , j , ρ . De la definición de la nueva 4-corriente j = ( C ρ , j ) comprobamos que es un cuadrivector. Esto se hace en mi RESPUESTA B para el caso de 1+1 dimensiones aquí: ¿Cómo demostramos que el jμ de 4 corrientes se transforma como xμ bajo la transformación de Lorentz? , votado negativo dos veces.

Estoy respondiendo esta pregunta como tarea, así que intentaré compartir aquí lo que estoy pensando:

Demos por sentado que al menos j m es un 4-vector. Entonces, volvamos a las ecuaciones de Maxwell, que en el calibre de Lorentz ( m A m = 0 ) son (estoy haciendo GR, así que estoy usando la métrica η m v = d i a gramo ( 1 , 1 , 1 , 1 ) , y hay un par de signos diferentes con respecto al de física de partículas, con suerte eso no debería interferir con el razonamiento)

A m = j m
Si j m es un cuadrivector, es la cantidad A m (y no solo A m ) que debe comportarse como un cuadrivector.

Como ya se señaló en algunas respuestas anteriores, como es un escalar de Lorentz, se sigue que A m es un cuadrivector, hasta una función con dalembertiano nulo: si A ~ m ( X ) es el 4-vector que satisface A ~ m = j m , y x m es cualquier colección de funciones que satisfacen x m = 0 para todos m , entonces la cantidad

A m = A ~ m + x m
satisface las ecuaciones de Maxwell y el requisito de que A m Sea un 4-vector. Esto aún no es suficiente para afirmar que A m es un vector de 4, pero la restricción adicional del calibre de Lorentz impone
0 = m A m = m A ~ m + m x m m A ~ m = m x m
así que de nuevo, la cantidad m x m debe comportarse como un escalar, lo que significa que el conjunto de funciones x m tiene un componente puro de 4 vectores x ~ m , y se define hasta una función h m , no necesariamente un vector de 4, pero con divergencia nula:
x m = x ~ m + h m , m h m = 0
Finalmente:
A m = A ~ m 4 v + x ~ m 4 v + h m
Cambiando el marco de referencia, A ~ m y x ~ m se transformará como 4-vectores en A ~ m y x ~ m , mientras h m en principio podría transformarse en algunos h m lo cual no es Λ m v h v , y que no satisface las necesidades m h m = 0 . Pero gracias a la invariancia de calibre, siempre podemos realizar una redefinición del campo. A m que cancela la 4-divergencia distinta de cero:
A m A m = A ~ m 4 v + x ~ m 4 v + h m + m Φ
m h m + m m Φ = 0 Φ = m h m
La función Φ siempre se puede determinar utilizando la función de Green para el dalembertiano. Todo en el lado izquierdo es un escalar, entonces h m solo puede ser un vector de 4 h ~ m más una parte sin divergencia:
h m = h ~ m + gramo m , m gramo m = 0
Así que el campo en este nuevo marco es
A m = A ~ m 4 v + x ~ m 4 v + h ~ m 4 v + m Φ 4 v + gramo m
Comparando los dos potenciales en los dos marcos:
A m = A ~ m + x ~ m 4 v + h m m h m = 0 A m = A ~ m + x ~ m + h ~ m + m Φ 4 v + gramo m m gramo m = 0 A m = A ~ m + x ~ m 4 v + gramo m m gramo m = 0 ( elegir  m Φ = h m )
En pocas palabras: elegir adecuadamente Φ podríamos cancelar h ~ m out y al menos hacer coincidir la parte de 4 vectores de los dos campos. Pero nunca podremos eliminar del campo la parte no vectorial, sin divergencia, que luego seguirá siendo globalmente un vector no cuaternario.

En mi humilde opinión, A m necesita ser definido para ser un 4-vector.

Quiero mostrar que la corriente de cuatro es de hecho un vector de cuatro, la prueba de los potenciales se deduce de las ecuaciones diferenciales como solución suficiente en lugar de necesaria, como respondió otro usuario.

X 0 ser las coordenadas espaciales del marco donde la densidad de carga ρ 0 está en reposo

X coordenadas espaciales de otro marco donde la densidad de carga es ρ X y alejándose con velocidad v X del resto del marco

γ = 1 1 v X 2 C 2

Entonces, debido a la invariancia de carga y la contracción de longitud ρ 0 d 3 X 0 = ρ X d 3 X = 1 γ ρ X d 3 X 0

ρ X = γ ρ 0

densidad actual j X = ρ X v X = γ ρ 0 v X

ahora el vector ( j X C , γ ρ 0 ) = ρ 0 ( γ v X C , γ ) es un cuatro vector porque ( γ v X C , γ ) es la velocidad propia en relatividad especial

Los cuatro vectores están definidos por leyes de transformación. Si los efectos físicos de A m en un marco son los mismos que los efectos físicos de Λ v m A v en un marco diferente - entonces se puede decir que A es un cuatro vector.

Los efectos físicos se pueden ver a partir de las ecuaciones de Maxwell, que tienen forma contravariante cuando se expresan en términos del tensor de intensidad de campo:

m F m v = 0

Ley de transformación de F implica la ley de transformación de cuatro vectores de A .

Consulte Tensor electromagnético para obtener más detalles.

Le agradecería mucho que me mostrara la forma en que se cumple la ley de transformación del tensor electromagnético F m v implica la ley de transformación de Lorentz de cuatro vectores de los 4 potenciales A = ( A , ϕ / C ) = ( A 1 , A 2 , A 3 , A 4 = ϕ / C ) ... (1)...
...(1)...Para simplificar, tome el caso de 1+1 dimensiones para mostrarme cómo probar las ecuaciones
(01a) A 1 = γ ( A 1 υ C A 4 ) = γ ( A 1 υ ϕ C 2 ) (01b) A 2 = A 2 (01c) A 3 = A 3 A 4 = γ ( A 4 υ C A 1 ) eso es (01d) ϕ = γ ( ϕ υ A 1 )
Muchas gracias de antemano.
Rechacé tu respuesta como incorrecta.

@thesundayscientist . No, el potencial de 4 de Maxwell A m no es un campo de 4 vectores válido (es decir, transformando bajo la representación fundamental de L o r ( 3 , 1 ) , o de manera equivalente, bajo el ( 1 / 2 , 1 / 2 ) representacion de SL ( 2 , C ) ), al menos no en la teoría cuántica de campos. La simetría de calibre estropea la ley de transformación correcta bajo un elemento Λ L o r ( 3 , 1 ) del campo de 4 potenciales cuantizado, como lo muestra S.Weinberg en las páginas 249-251 del primer volumen de su famoso tratado sobre QFT. Sin embargo, el llamado tensor de Faraday de la electrodinámica F m v es un campo de tensor de Lorentz antisimétrico válido, porque el rotacional elimina el término de calibre. Esto también lo aclara Weinberg, op.cit.

EDITAR: Aquí hay un conflicto de definiciones que no se aborda. Un campo de 4 vectores en la teoría clásica de campos es [ignorando por completo todas las diff.geom. aspectos] una asignación suave de una tupla de 4 funciones a cada punto en el espacio-tiempo, con la propiedad de que, bajo un cambio de observadores inerciales cuyas coordenadas de sus respectivos puntos se transforman mediante una transformación de Lorentz restringida, se transforman covariantemente: V m ( X ) V m ( X ) = Λ   v m V v ( X ) . Bajo esta definición, se puede decir que el 4 potencial de Maxwell es un 4 vector . Un campo de 4 vectores en la teoría cuántica de campos se define según los axiomas de Wightman como: la 4-tupla de las distribuciones valoradas por el operador del espacio de Fock en el espacio-tiempo plano de Minkowski cuya covarianza bajo ISL ( 2 , C ) está dado por Lopuszanski. Lo que Wightman está diciendo es que el potencial electromagnético cuantificado, precisamente debido a la invariancia de calibre que proviene de la falta de masa del campo clásico, no puede ser covariante bajo ISL ( 2 , C ) en el sentido de la ec. (4.1.82a) del libro de Lopuszanski sobre QFT axiomático

.

El hecho de que tenga simetría de calibre no significa que no sea un 4-vector válido. Un vector de 4 se define por cómo se transforma, no por su singularidad.
Eso es lo que Weinberg está mostrando. Un campo de 4 vectores de masa cero no es covariante bajo una representación unitaria del subgrupo del grupo de cobertura universal del grupo de Poincaré restringido isomorfo al grupo de cobertura universal del grupo de Lorentz restringido. Entonces, el potencial 4 de Maxwell no es un vector 4 bajo I S L ( 2 , C ) .
¿Quizás deberías cambiar la respuesta a solo la parte de edición? Parece bastante claro que el OP estaba hablando de la teoría de campo clásica.
Es molesto cuando las personas responden preguntas simples con respuestas demasiado complicadas solo para presumir aquí.
Rechacé tu respuesta porque: " Es molesto cuando las personas responden preguntas simples con respuestas demasiado complicadas... "